How to prove $E[g(x)] = int_0^infty g'(x)S(x) dx$ Announcing the arrival of Valued Associate #679: Cesar Manara Planned maintenance scheduled April 23, 2019 at 23:30 UTC (7:30pm US/Eastern)Is it true that $limlimits_xtoinftyx·P[X>x]=0$?Proof of $textTVaR_p(X)$ and $textVaR_u(X)$ relationshipShow that if X is a continuous random variable on $[b,infty)$ then $mathrmE[X]=b+int_b^infty(1- F(x))dx $For a pdf $f(x)$, how can we prove that $int_-infty^infty x,f(x),dx=int_-infty^infty F(xgeq t),dt$?Prove the following equality to show a relationship between Poisson and Gamma Random VariablesHow can I prove that expectation of conditional random variable?Peculiar problem about characteristic function and density function.With scant information, how to prove this probability limit tends to zero?How to prove that a conditional pdf sums to 1?How to calculate a joint pdf by convolutionFind the conditional density function and expectation of $Y$ given $X$ when $f(x,y) = lambda^2e^-lambda y$ and $f(x,y) = xe^-x(y+1)$

C's equality operator on converted pointers

Does "shooting for effect" have contradictory meanings in different areas?

Did any compiler fully use 80-bit floating point?

Why are my pictures showing a dark band on one edge?

Draw 4 of the same figure in the same tikzpicture

What is the meaning of 'breadth' in breadth first search?

Is it fair for a professor to grade us on the possession of past papers?

Central Vacuuming: Is it worth it, and how does it compare to normal vacuuming?

Intuitive explanation of the rank-nullity theorem

What does Turing mean by this statement?

Why weren't discrete x86 CPUs ever used in game hardware?

Misunderstanding of Sylow theory

Is there public access to the Meteor Crater in Arizona?

What are the discoveries that have been possible with the rejection of positivism?

A letter with no particular backstory

Dyck paths with extra diagonals from valleys (Laser construction)

Would it be easier to apply for a UK visa if there is a host family to sponsor for you in going there?

Karn the great creator - 'card from outside the game' in sealed

Do wooden building fires get hotter than 600°C?

How many morphisms from 1 to 1+1 can there be?

Co-worker has annoying ringtone

Drawing spherical mirrors

What makes a man succeed?

How to identify unknown coordinate type and convert to lat/lon?



How to prove $E[g(x)] = int_0^infty g'(x)S(x) dx$



Announcing the arrival of Valued Associate #679: Cesar Manara
Planned maintenance scheduled April 23, 2019 at 23:30 UTC (7:30pm US/Eastern)Is it true that $limlimits_xtoinftyx·P[X>x]=0$?Proof of $textTVaR_p(X)$ and $textVaR_u(X)$ relationshipShow that if X is a continuous random variable on $[b,infty)$ then $mathrmE[X]=b+int_b^infty(1- F(x))dx $For a pdf $f(x)$, how can we prove that $int_-infty^infty x,f(x),dx=int_-infty^infty F(xgeq t),dt$?Prove the following equality to show a relationship between Poisson and Gamma Random VariablesHow can I prove that expectation of conditional random variable?Peculiar problem about characteristic function and density function.With scant information, how to prove this probability limit tends to zero?How to prove that a conditional pdf sums to 1?How to calculate a joint pdf by convolutionFind the conditional density function and expectation of $Y$ given $X$ when $f(x,y) = lambda^2e^-lambda y$ and $f(x,y) = xe^-x(y+1)$










1












$begingroup$


Given that $E[g(X)] = int_-infty^infty g(x)f(x) dx$, how to prove $E[g(X)] = int_0^infty g'(x)S(x)dx$, where $S(x) = 1-F(x)$?



By integration by parts, I can get the following:
beginalign
E[g(X)] &= int_-infty^infty g(x)f(x) dx \
&= int_-infty^infty g(x)F'(x) dx \
&= g(x)F(x)|_-infty^infty - int_-infty^infty g'(x)F(x) dx \
&= g(x)F(x)|_-infty^infty + int_-infty^infty g'(x)[1-S(x)] dx \
&= g(x)F(x)|_-infty^infty - int_-infty^infty g'(x)dx +int_-infty^infty g'(x)S(x) dx\
endalign



Then I am stuck.










share|cite|improve this question









$endgroup$
















    1












    $begingroup$


    Given that $E[g(X)] = int_-infty^infty g(x)f(x) dx$, how to prove $E[g(X)] = int_0^infty g'(x)S(x)dx$, where $S(x) = 1-F(x)$?



    By integration by parts, I can get the following:
    beginalign
    E[g(X)] &= int_-infty^infty g(x)f(x) dx \
    &= int_-infty^infty g(x)F'(x) dx \
    &= g(x)F(x)|_-infty^infty - int_-infty^infty g'(x)F(x) dx \
    &= g(x)F(x)|_-infty^infty + int_-infty^infty g'(x)[1-S(x)] dx \
    &= g(x)F(x)|_-infty^infty - int_-infty^infty g'(x)dx +int_-infty^infty g'(x)S(x) dx\
    endalign



    Then I am stuck.










    share|cite|improve this question









    $endgroup$














      1












      1








      1


      2



      $begingroup$


      Given that $E[g(X)] = int_-infty^infty g(x)f(x) dx$, how to prove $E[g(X)] = int_0^infty g'(x)S(x)dx$, where $S(x) = 1-F(x)$?



      By integration by parts, I can get the following:
      beginalign
      E[g(X)] &= int_-infty^infty g(x)f(x) dx \
      &= int_-infty^infty g(x)F'(x) dx \
      &= g(x)F(x)|_-infty^infty - int_-infty^infty g'(x)F(x) dx \
      &= g(x)F(x)|_-infty^infty + int_-infty^infty g'(x)[1-S(x)] dx \
      &= g(x)F(x)|_-infty^infty - int_-infty^infty g'(x)dx +int_-infty^infty g'(x)S(x) dx\
      endalign



      Then I am stuck.










      share|cite|improve this question









      $endgroup$




      Given that $E[g(X)] = int_-infty^infty g(x)f(x) dx$, how to prove $E[g(X)] = int_0^infty g'(x)S(x)dx$, where $S(x) = 1-F(x)$?



      By integration by parts, I can get the following:
      beginalign
      E[g(X)] &= int_-infty^infty g(x)f(x) dx \
      &= int_-infty^infty g(x)F'(x) dx \
      &= g(x)F(x)|_-infty^infty - int_-infty^infty g'(x)F(x) dx \
      &= g(x)F(x)|_-infty^infty + int_-infty^infty g'(x)[1-S(x)] dx \
      &= g(x)F(x)|_-infty^infty - int_-infty^infty g'(x)dx +int_-infty^infty g'(x)S(x) dx\
      endalign



      Then I am stuck.







      probability






      share|cite|improve this question













      share|cite|improve this question











      share|cite|improve this question




      share|cite|improve this question










      asked Apr 2 at 0:18









      JangoJango

      1839




      1839




















          2 Answers
          2






          active

          oldest

          votes


















          3












          $begingroup$

          I think what you are trying to prove is not exactly correct. I will work through the problem and show the correct answer at the end.



          You need to split the integral into two, one for the region $xge 0$ and one for $xle 0$.



          In the positive region, instead of writing $f(x)=F'(x)$ and integrating by parts, you need to use $f=(F(x)-1)'$. This is the only choice which ensures that the resulting integral actually exists; note that $F(x)-1$ goes to zero as $xto+infty$, but $F(x)$ does not.



          beginalign
          int_0^infty g(x)f(x),dx
          &=int_0^infty g(x)[F(x)-1]',dx
          \&=g(x)[F(x)-1]Big|^infty_0-int_0^infty g'(s)[F(x)-1],dx
          \&=underbraceBig(lim_xtoinftyg(x)[F(x)-1]Big)_=0-g(0)[F(0)-1]+int_0^infty g'(x)S(x),dx
          endalign

          It takes some doing, but you can show that that limit actually is zero, as long as $E[g(X)]$ is finite. See Is it true that $limlimits_xtoinftyx·P[X>x]=0$? for some inspiration. Edit: Actually, I am not quite sure this is true without some additional assumptions on $g.$ It is true as long as $g$ is either bounded or monotonic.



          For the negative region, you do want to use $F(x)$ as the antiderivate of $f(x)$, because $lim_xto-inftyF(x)=0$.
          beginalign
          int_-infty^0 g(x)f(x),dx
          &=int_-infty^0 g(x)F(x)',dx
          \&=g(x)F(x)Big|^0_-infty-int_-infty^0 g'(s)F(x),dx
          \&=g(0)F(0)-underbraceBig(lim_xto-inftyg(x)F(x)Big)_=0-int_-infty^0 g'(x)F(x),dx
          endalign

          Putting this all together, we get
          $$
          bbox[7px,border:2px solid red]int_-infty^infty g(x)f(x),dx=g(0)+int_0^infty g'(x)S(x),dx-int_-infty^0g'(x)F(x),dx
          $$

          This is correct for any function $g$ such that $E[g(X)]$ is finite. Edit: Well, as long as $g$ is either bounded or monotonic.



          If we make further assumptions about $g$, we can write this more nicely. Assuming $g(-infty):=lim_xto-inftyg(x)$ exists, you would have
          $$
          g(0)=g(-infty)+int_-infty^0 g'(x),dx
          $$

          so
          $$
          bbox[7px,border:2px solid green]int_-infty^infty g(x)f(x),dx=g(-infty)+int_-infty^infty g'(x)S(x),dx
          $$

          You cannot in general avoid the presence of some constant $g(a)$. This is because the $int g'(x)S(x),dx$ part of the formula does not change when $g$ is shifted by a constant (this does not affect $g'$), but $E[g(x)]$ does change when $g$ is shifted by a constant.






          share|cite|improve this answer











          $endgroup$












          • $begingroup$
            Thank you very much. I did not think it is so difficult since I read this from "Loss Model" other than analysis. Anyway, your work is clear and easy to understand. Thanks.
            $endgroup$
            – Jango
            Apr 2 at 14:34










          • $begingroup$
            @Jango Can I ask where this appears in the book?
            $endgroup$
            – Mike Earnest
            Apr 2 at 14:48


















          1












          $begingroup$

          Write $1-F(x)=int_(x,infty) dmu(y)$ where $mu$ is the measure corresponding to $F$. Now apply Fubini/Tonelli Theorem to the right hand side.






          share|cite|improve this answer









          $endgroup$













            Your Answer








            StackExchange.ready(function()
            var channelOptions =
            tags: "".split(" "),
            id: "69"
            ;
            initTagRenderer("".split(" "), "".split(" "), channelOptions);

            StackExchange.using("externalEditor", function()
            // Have to fire editor after snippets, if snippets enabled
            if (StackExchange.settings.snippets.snippetsEnabled)
            StackExchange.using("snippets", function()
            createEditor();
            );

            else
            createEditor();

            );

            function createEditor()
            StackExchange.prepareEditor(
            heartbeatType: 'answer',
            autoActivateHeartbeat: false,
            convertImagesToLinks: true,
            noModals: true,
            showLowRepImageUploadWarning: true,
            reputationToPostImages: 10,
            bindNavPrevention: true,
            postfix: "",
            imageUploader:
            brandingHtml: "Powered by u003ca class="icon-imgur-white" href="https://imgur.com/"u003eu003c/au003e",
            contentPolicyHtml: "User contributions licensed under u003ca href="https://creativecommons.org/licenses/by-sa/3.0/"u003ecc by-sa 3.0 with attribution requiredu003c/au003e u003ca href="https://stackoverflow.com/legal/content-policy"u003e(content policy)u003c/au003e",
            allowUrls: true
            ,
            noCode: true, onDemand: true,
            discardSelector: ".discard-answer"
            ,immediatelyShowMarkdownHelp:true
            );



            );













            draft saved

            draft discarded


















            StackExchange.ready(
            function ()
            StackExchange.openid.initPostLogin('.new-post-login', 'https%3a%2f%2fmath.stackexchange.com%2fquestions%2f3171308%2fhow-to-prove-egx-int-0-infty-gxsx-dx%23new-answer', 'question_page');

            );

            Post as a guest















            Required, but never shown

























            2 Answers
            2






            active

            oldest

            votes








            2 Answers
            2






            active

            oldest

            votes









            active

            oldest

            votes






            active

            oldest

            votes









            3












            $begingroup$

            I think what you are trying to prove is not exactly correct. I will work through the problem and show the correct answer at the end.



            You need to split the integral into two, one for the region $xge 0$ and one for $xle 0$.



            In the positive region, instead of writing $f(x)=F'(x)$ and integrating by parts, you need to use $f=(F(x)-1)'$. This is the only choice which ensures that the resulting integral actually exists; note that $F(x)-1$ goes to zero as $xto+infty$, but $F(x)$ does not.



            beginalign
            int_0^infty g(x)f(x),dx
            &=int_0^infty g(x)[F(x)-1]',dx
            \&=g(x)[F(x)-1]Big|^infty_0-int_0^infty g'(s)[F(x)-1],dx
            \&=underbraceBig(lim_xtoinftyg(x)[F(x)-1]Big)_=0-g(0)[F(0)-1]+int_0^infty g'(x)S(x),dx
            endalign

            It takes some doing, but you can show that that limit actually is zero, as long as $E[g(X)]$ is finite. See Is it true that $limlimits_xtoinftyx·P[X>x]=0$? for some inspiration. Edit: Actually, I am not quite sure this is true without some additional assumptions on $g.$ It is true as long as $g$ is either bounded or monotonic.



            For the negative region, you do want to use $F(x)$ as the antiderivate of $f(x)$, because $lim_xto-inftyF(x)=0$.
            beginalign
            int_-infty^0 g(x)f(x),dx
            &=int_-infty^0 g(x)F(x)',dx
            \&=g(x)F(x)Big|^0_-infty-int_-infty^0 g'(s)F(x),dx
            \&=g(0)F(0)-underbraceBig(lim_xto-inftyg(x)F(x)Big)_=0-int_-infty^0 g'(x)F(x),dx
            endalign

            Putting this all together, we get
            $$
            bbox[7px,border:2px solid red]int_-infty^infty g(x)f(x),dx=g(0)+int_0^infty g'(x)S(x),dx-int_-infty^0g'(x)F(x),dx
            $$

            This is correct for any function $g$ such that $E[g(X)]$ is finite. Edit: Well, as long as $g$ is either bounded or monotonic.



            If we make further assumptions about $g$, we can write this more nicely. Assuming $g(-infty):=lim_xto-inftyg(x)$ exists, you would have
            $$
            g(0)=g(-infty)+int_-infty^0 g'(x),dx
            $$

            so
            $$
            bbox[7px,border:2px solid green]int_-infty^infty g(x)f(x),dx=g(-infty)+int_-infty^infty g'(x)S(x),dx
            $$

            You cannot in general avoid the presence of some constant $g(a)$. This is because the $int g'(x)S(x),dx$ part of the formula does not change when $g$ is shifted by a constant (this does not affect $g'$), but $E[g(x)]$ does change when $g$ is shifted by a constant.






            share|cite|improve this answer











            $endgroup$












            • $begingroup$
              Thank you very much. I did not think it is so difficult since I read this from "Loss Model" other than analysis. Anyway, your work is clear and easy to understand. Thanks.
              $endgroup$
              – Jango
              Apr 2 at 14:34










            • $begingroup$
              @Jango Can I ask where this appears in the book?
              $endgroup$
              – Mike Earnest
              Apr 2 at 14:48















            3












            $begingroup$

            I think what you are trying to prove is not exactly correct. I will work through the problem and show the correct answer at the end.



            You need to split the integral into two, one for the region $xge 0$ and one for $xle 0$.



            In the positive region, instead of writing $f(x)=F'(x)$ and integrating by parts, you need to use $f=(F(x)-1)'$. This is the only choice which ensures that the resulting integral actually exists; note that $F(x)-1$ goes to zero as $xto+infty$, but $F(x)$ does not.



            beginalign
            int_0^infty g(x)f(x),dx
            &=int_0^infty g(x)[F(x)-1]',dx
            \&=g(x)[F(x)-1]Big|^infty_0-int_0^infty g'(s)[F(x)-1],dx
            \&=underbraceBig(lim_xtoinftyg(x)[F(x)-1]Big)_=0-g(0)[F(0)-1]+int_0^infty g'(x)S(x),dx
            endalign

            It takes some doing, but you can show that that limit actually is zero, as long as $E[g(X)]$ is finite. See Is it true that $limlimits_xtoinftyx·P[X>x]=0$? for some inspiration. Edit: Actually, I am not quite sure this is true without some additional assumptions on $g.$ It is true as long as $g$ is either bounded or monotonic.



            For the negative region, you do want to use $F(x)$ as the antiderivate of $f(x)$, because $lim_xto-inftyF(x)=0$.
            beginalign
            int_-infty^0 g(x)f(x),dx
            &=int_-infty^0 g(x)F(x)',dx
            \&=g(x)F(x)Big|^0_-infty-int_-infty^0 g'(s)F(x),dx
            \&=g(0)F(0)-underbraceBig(lim_xto-inftyg(x)F(x)Big)_=0-int_-infty^0 g'(x)F(x),dx
            endalign

            Putting this all together, we get
            $$
            bbox[7px,border:2px solid red]int_-infty^infty g(x)f(x),dx=g(0)+int_0^infty g'(x)S(x),dx-int_-infty^0g'(x)F(x),dx
            $$

            This is correct for any function $g$ such that $E[g(X)]$ is finite. Edit: Well, as long as $g$ is either bounded or monotonic.



            If we make further assumptions about $g$, we can write this more nicely. Assuming $g(-infty):=lim_xto-inftyg(x)$ exists, you would have
            $$
            g(0)=g(-infty)+int_-infty^0 g'(x),dx
            $$

            so
            $$
            bbox[7px,border:2px solid green]int_-infty^infty g(x)f(x),dx=g(-infty)+int_-infty^infty g'(x)S(x),dx
            $$

            You cannot in general avoid the presence of some constant $g(a)$. This is because the $int g'(x)S(x),dx$ part of the formula does not change when $g$ is shifted by a constant (this does not affect $g'$), but $E[g(x)]$ does change when $g$ is shifted by a constant.






            share|cite|improve this answer











            $endgroup$












            • $begingroup$
              Thank you very much. I did not think it is so difficult since I read this from "Loss Model" other than analysis. Anyway, your work is clear and easy to understand. Thanks.
              $endgroup$
              – Jango
              Apr 2 at 14:34










            • $begingroup$
              @Jango Can I ask where this appears in the book?
              $endgroup$
              – Mike Earnest
              Apr 2 at 14:48













            3












            3








            3





            $begingroup$

            I think what you are trying to prove is not exactly correct. I will work through the problem and show the correct answer at the end.



            You need to split the integral into two, one for the region $xge 0$ and one for $xle 0$.



            In the positive region, instead of writing $f(x)=F'(x)$ and integrating by parts, you need to use $f=(F(x)-1)'$. This is the only choice which ensures that the resulting integral actually exists; note that $F(x)-1$ goes to zero as $xto+infty$, but $F(x)$ does not.



            beginalign
            int_0^infty g(x)f(x),dx
            &=int_0^infty g(x)[F(x)-1]',dx
            \&=g(x)[F(x)-1]Big|^infty_0-int_0^infty g'(s)[F(x)-1],dx
            \&=underbraceBig(lim_xtoinftyg(x)[F(x)-1]Big)_=0-g(0)[F(0)-1]+int_0^infty g'(x)S(x),dx
            endalign

            It takes some doing, but you can show that that limit actually is zero, as long as $E[g(X)]$ is finite. See Is it true that $limlimits_xtoinftyx·P[X>x]=0$? for some inspiration. Edit: Actually, I am not quite sure this is true without some additional assumptions on $g.$ It is true as long as $g$ is either bounded or monotonic.



            For the negative region, you do want to use $F(x)$ as the antiderivate of $f(x)$, because $lim_xto-inftyF(x)=0$.
            beginalign
            int_-infty^0 g(x)f(x),dx
            &=int_-infty^0 g(x)F(x)',dx
            \&=g(x)F(x)Big|^0_-infty-int_-infty^0 g'(s)F(x),dx
            \&=g(0)F(0)-underbraceBig(lim_xto-inftyg(x)F(x)Big)_=0-int_-infty^0 g'(x)F(x),dx
            endalign

            Putting this all together, we get
            $$
            bbox[7px,border:2px solid red]int_-infty^infty g(x)f(x),dx=g(0)+int_0^infty g'(x)S(x),dx-int_-infty^0g'(x)F(x),dx
            $$

            This is correct for any function $g$ such that $E[g(X)]$ is finite. Edit: Well, as long as $g$ is either bounded or monotonic.



            If we make further assumptions about $g$, we can write this more nicely. Assuming $g(-infty):=lim_xto-inftyg(x)$ exists, you would have
            $$
            g(0)=g(-infty)+int_-infty^0 g'(x),dx
            $$

            so
            $$
            bbox[7px,border:2px solid green]int_-infty^infty g(x)f(x),dx=g(-infty)+int_-infty^infty g'(x)S(x),dx
            $$

            You cannot in general avoid the presence of some constant $g(a)$. This is because the $int g'(x)S(x),dx$ part of the formula does not change when $g$ is shifted by a constant (this does not affect $g'$), but $E[g(x)]$ does change when $g$ is shifted by a constant.






            share|cite|improve this answer











            $endgroup$



            I think what you are trying to prove is not exactly correct. I will work through the problem and show the correct answer at the end.



            You need to split the integral into two, one for the region $xge 0$ and one for $xle 0$.



            In the positive region, instead of writing $f(x)=F'(x)$ and integrating by parts, you need to use $f=(F(x)-1)'$. This is the only choice which ensures that the resulting integral actually exists; note that $F(x)-1$ goes to zero as $xto+infty$, but $F(x)$ does not.



            beginalign
            int_0^infty g(x)f(x),dx
            &=int_0^infty g(x)[F(x)-1]',dx
            \&=g(x)[F(x)-1]Big|^infty_0-int_0^infty g'(s)[F(x)-1],dx
            \&=underbraceBig(lim_xtoinftyg(x)[F(x)-1]Big)_=0-g(0)[F(0)-1]+int_0^infty g'(x)S(x),dx
            endalign

            It takes some doing, but you can show that that limit actually is zero, as long as $E[g(X)]$ is finite. See Is it true that $limlimits_xtoinftyx·P[X>x]=0$? for some inspiration. Edit: Actually, I am not quite sure this is true without some additional assumptions on $g.$ It is true as long as $g$ is either bounded or monotonic.



            For the negative region, you do want to use $F(x)$ as the antiderivate of $f(x)$, because $lim_xto-inftyF(x)=0$.
            beginalign
            int_-infty^0 g(x)f(x),dx
            &=int_-infty^0 g(x)F(x)',dx
            \&=g(x)F(x)Big|^0_-infty-int_-infty^0 g'(s)F(x),dx
            \&=g(0)F(0)-underbraceBig(lim_xto-inftyg(x)F(x)Big)_=0-int_-infty^0 g'(x)F(x),dx
            endalign

            Putting this all together, we get
            $$
            bbox[7px,border:2px solid red]int_-infty^infty g(x)f(x),dx=g(0)+int_0^infty g'(x)S(x),dx-int_-infty^0g'(x)F(x),dx
            $$

            This is correct for any function $g$ such that $E[g(X)]$ is finite. Edit: Well, as long as $g$ is either bounded or monotonic.



            If we make further assumptions about $g$, we can write this more nicely. Assuming $g(-infty):=lim_xto-inftyg(x)$ exists, you would have
            $$
            g(0)=g(-infty)+int_-infty^0 g'(x),dx
            $$

            so
            $$
            bbox[7px,border:2px solid green]int_-infty^infty g(x)f(x),dx=g(-infty)+int_-infty^infty g'(x)S(x),dx
            $$

            You cannot in general avoid the presence of some constant $g(a)$. This is because the $int g'(x)S(x),dx$ part of the formula does not change when $g$ is shifted by a constant (this does not affect $g'$), but $E[g(x)]$ does change when $g$ is shifted by a constant.







            share|cite|improve this answer














            share|cite|improve this answer



            share|cite|improve this answer








            edited Apr 2 at 4:54

























            answered Apr 2 at 4:27









            Mike EarnestMike Earnest

            28.2k22152




            28.2k22152











            • $begingroup$
              Thank you very much. I did not think it is so difficult since I read this from "Loss Model" other than analysis. Anyway, your work is clear and easy to understand. Thanks.
              $endgroup$
              – Jango
              Apr 2 at 14:34










            • $begingroup$
              @Jango Can I ask where this appears in the book?
              $endgroup$
              – Mike Earnest
              Apr 2 at 14:48
















            • $begingroup$
              Thank you very much. I did not think it is so difficult since I read this from "Loss Model" other than analysis. Anyway, your work is clear and easy to understand. Thanks.
              $endgroup$
              – Jango
              Apr 2 at 14:34










            • $begingroup$
              @Jango Can I ask where this appears in the book?
              $endgroup$
              – Mike Earnest
              Apr 2 at 14:48















            $begingroup$
            Thank you very much. I did not think it is so difficult since I read this from "Loss Model" other than analysis. Anyway, your work is clear and easy to understand. Thanks.
            $endgroup$
            – Jango
            Apr 2 at 14:34




            $begingroup$
            Thank you very much. I did not think it is so difficult since I read this from "Loss Model" other than analysis. Anyway, your work is clear and easy to understand. Thanks.
            $endgroup$
            – Jango
            Apr 2 at 14:34












            $begingroup$
            @Jango Can I ask where this appears in the book?
            $endgroup$
            – Mike Earnest
            Apr 2 at 14:48




            $begingroup$
            @Jango Can I ask where this appears in the book?
            $endgroup$
            – Mike Earnest
            Apr 2 at 14:48











            1












            $begingroup$

            Write $1-F(x)=int_(x,infty) dmu(y)$ where $mu$ is the measure corresponding to $F$. Now apply Fubini/Tonelli Theorem to the right hand side.






            share|cite|improve this answer









            $endgroup$

















              1












              $begingroup$

              Write $1-F(x)=int_(x,infty) dmu(y)$ where $mu$ is the measure corresponding to $F$. Now apply Fubini/Tonelli Theorem to the right hand side.






              share|cite|improve this answer









              $endgroup$















                1












                1








                1





                $begingroup$

                Write $1-F(x)=int_(x,infty) dmu(y)$ where $mu$ is the measure corresponding to $F$. Now apply Fubini/Tonelli Theorem to the right hand side.






                share|cite|improve this answer









                $endgroup$



                Write $1-F(x)=int_(x,infty) dmu(y)$ where $mu$ is the measure corresponding to $F$. Now apply Fubini/Tonelli Theorem to the right hand side.







                share|cite|improve this answer












                share|cite|improve this answer



                share|cite|improve this answer










                answered Apr 2 at 0:22









                Kavi Rama MurthyKavi Rama Murthy

                76.1k53370




                76.1k53370



























                    draft saved

                    draft discarded
















































                    Thanks for contributing an answer to Mathematics Stack Exchange!


                    • Please be sure to answer the question. Provide details and share your research!

                    But avoid


                    • Asking for help, clarification, or responding to other answers.

                    • Making statements based on opinion; back them up with references or personal experience.

                    Use MathJax to format equations. MathJax reference.


                    To learn more, see our tips on writing great answers.




                    draft saved


                    draft discarded














                    StackExchange.ready(
                    function ()
                    StackExchange.openid.initPostLogin('.new-post-login', 'https%3a%2f%2fmath.stackexchange.com%2fquestions%2f3171308%2fhow-to-prove-egx-int-0-infty-gxsx-dx%23new-answer', 'question_page');

                    );

                    Post as a guest















                    Required, but never shown





















































                    Required, but never shown














                    Required, but never shown












                    Required, but never shown







                    Required, but never shown

































                    Required, but never shown














                    Required, but never shown












                    Required, but never shown







                    Required, but never shown







                    Popular posts from this blog

                    Triangular numbers and gcdProving sum of a set is $0 pmod n$ if $n$ is odd, or $fracn2 pmod n$ if $n$ is even?Is greatest common divisor of two numbers really their smallest linear combination?GCD, LCM RelationshipProve a set of nonnegative integers with greatest common divisor 1 and closed under addition has all but finite many nonnegative integers.all pairs of a and b in an equation containing gcdTriangular Numbers Modulo $k$ - Hit All Values?Understanding the Existence and Uniqueness of the GCDGCD and LCM with logical symbolsThe greatest common divisor of two positive integers less than 100 is equal to 3. Their least common multiple is twelve times one of the integers.Suppose that for all integers $x$, $x|a$ and $x|b$ if and only if $x|c$. Then $c = gcd(a,b)$Which is the gcd of 2 numbers which are multiplied and the result is 600000?

                    Ingelân Ynhâld Etymology | Geografy | Skiednis | Polityk en bestjoer | Ekonomy | Demografy | Kultuer | Klimaat | Sjoch ek | Keppelings om utens | Boarnen, noaten en referinsjes Navigaasjemenuwww.gov.ukOffisjele webside fan it regear fan it Feriene KeninkrykOffisjele webside fan it Britske FerkearsburoNederlânsktalige ynformaasje fan it Britske FerkearsburoOffisjele webside fan English Heritage, de organisaasje dy't him ynset foar it behâld fan it Ingelske kultuergoedYnwennertallen fan alle Britske stêden út 'e folkstelling fan 2011Notes en References, op dizze sideEngland

                    Հադիս Բովանդակություն Անվանում և նշանակություն | Դասակարգում | Աղբյուրներ | Նավարկման ցանկ